0 Daumen
225 Aufrufe

Aufgabe: Das Polynom f= X^4 + X^3 + X^2 + X + 1 ist irreduziebel ∈ ℚ [X].

Geben Sie das Inverse von x , wobei x= X+f(x) ∈ L und y = x^3 + 2x^2 + 3 im Körper L an! L=ℚ(x)/(f)

Problem/Ansatz:

Mir ist nicht genau klar wie man das machen soll und wie genau ich das aufschreiben soll, ich wäre euch sehr dankbar wenn ihr mir helfen könntet das zu verstehen ..

Avatar von

Kann mir hier wirklich keiner weiter helfen? :(

Du bestimmst mit dem erweiterten euklidischen Algorithmus eine Darstellung der Form:

$$ \operatorname{ggT}(g, f) = \underbrace{q}_{\in \mathbb Q^*} = s \cdot g + t \cdot f $$

mit \( g = X^3 + 2X^2 + 3 \), \( f = X^4 + X^3 + X^2 + X + 1 \) und \( s, t \in \mathbb Q [X] \).

Dann folgt \( 1 = q^{-1} s g + q^{-1} t f \)

Und somit modulo \( (f) \): \( [1] = [q^{-1} s] \cdot [g] \).

D.h. das Inverse von \( [g] \) ist also gerade \( [q^{-1} s] \).

Der zweite Summand entfällt wegen \( [f]= 0 \).

Du kannst deine Rechnungen selbstständig mit einem CAS überprüfen. Z.B. mit SAGE

# Erweiterter euklidischer Algorithmus
def eea(a,b,quo=lambda a,b:a//b):
  r0 = a; r1 = b
  s0 = 1; s1 = 0
  t0 = 0; t1 = 1

  while r1 != 0:
      q = quo(r0, r1)
      r0, r1 = r1, r0 - q * r1
      s0, s1 = s1, s0 - q * s1
      t0, t1 = t1, t0 - q * t1

  return r0, s0, t0

###

K = QQ # K = Körper der rationalen Zahlen
R = PolynomialRing(K, 'X') # R = Q[X]
X = R.gen()

f = X^4 + X^3 + X^2 + X + 1
g = X^3 + 2 * X^2 + 3

q, s, t = eea(f, g)

print(q)
print(s)
print(t)

unter https://sagecell.sagemath.org/ kanst du den Code ohne Installation von SAGE ausführen.

Ein anderes Problem?

Stell deine Frage

Ähnliche Fragen

Willkommen bei der Mathelounge! Stell deine Frage einfach und kostenlos

x
Made by a lovely community